Solving an Indeterminate Limit with L'Hospital's Rule

  • Thread starter Thread starter student85
  • Start date Start date
  • Tags Tags
    Limit
Click For Summary
The limit in question is lim (lnx)^3 / x as x approaches infinity. Applying L'Hospital's Rule is necessary due to the indeterminate form. The solution requires multiple applications of L'Hospital's Rule, specifically four times, to resolve the indeterminate nature of the limit. The process can become complex and repetitive, but it ultimately leads to a definitive answer. Understanding the repeated application of L'Hospital's Rule is key to solving this limit.
student85
Messages
138
Reaction score
0

Homework Statement


find the limit:

lim (lnx)^3 / x
x-->(infinity)


Homework Equations


It's a L´hospital homework so that should be what I need to use.


The Attempt at a Solution


However, when using L'Hospital, I get nowhere since I keep getting indeterminations.
Help is appreciated.
 
Physics news on Phys.org
You'll have to repeatedly use L'Hopital until you get to a non-indeterminant. Takes 4 times, I think.
 
damn, it gets pretty nasty...
Thanks.
 
Nasty how? It's just a little repetitious.
 
Question: A clock's minute hand has length 4 and its hour hand has length 3. What is the distance between the tips at the moment when it is increasing most rapidly?(Putnam Exam Question) Answer: Making assumption that both the hands moves at constant angular velocities, the answer is ## \sqrt{7} .## But don't you think this assumption is somewhat doubtful and wrong?

Similar threads

  • · Replies 10 ·
Replies
10
Views
2K
Replies
6
Views
2K
  • · Replies 8 ·
Replies
8
Views
1K
  • · Replies 5 ·
Replies
5
Views
2K
  • · Replies 2 ·
Replies
2
Views
1K
Replies
4
Views
2K
Replies
10
Views
2K
Replies
35
Views
4K
  • · Replies 22 ·
Replies
22
Views
4K
  • · Replies 5 ·
Replies
5
Views
2K